Extrema mit Nebenbedingung

Neue Frage »

HBX8X Auf diesen Beitrag antworten »
Extrema mit Nebenbedingung
Guten Abend matheboard.de,

ich setze mich gerade mit einer Extremwertaufgabe mit Nebenbedingung auseinander. Leider bereitet mir diese Aufgabe etwas Probleme. Aufgabenstellung:

,

Nebenbedingung:

(a) Bestimmen Sie alle kritischen Punkte xi von f bezüglich M.

Meine-Idee:

Lagrange Funktion lautet:



So, nun muss ich das LGS lösen: Wie bestimm ich die Lösungsmenge und welche Fallunterscheidung wäre ratsam? Ich kann in der ersten Komponente des gradienten 2x ausklammern demnach wird 1. Fall wohl x=0 sein. Was ist aber mit xy^1=-1. Wie geh ich nun vor (falls das bisherige richtig ist).

Gruß.
klarsoweit Auf diesen Beitrag antworten »
RE: Extrema mit Nebenbedingung
Zitat:
Original von HBX8X
Was ist aber mit xy^1=-1.

Hm. Ich weiß jetzt nicht, wo du das her hast. Wie dem auch sei, aus der 1. Gleichung kannst du für x ungleich Null eine Bedingung für lambda herleiten.
HBX8X Auf diesen Beitrag antworten »
RE: Extrema mit Nebenbedingung
Zitat:
Original von klarsoweit
Zitat:
Original von HBX8X
Was ist aber mit xy^1=-1.

Hm. Ich weiß jetzt nicht, wo du das her hast. Wie dem auch sei, aus der 1. Gleichung kannst du für x ungleich Null eine Bedingung für lambda herleiten.


Ich hatte die erste Gleichung ausgeklammert, also die 2x. Demnach kann ich doch den Satzd es Nullprodukts anwenden, da x=0 ist (1. Fall). Also meinst du das der zweite Fall x ungleich 0 ist. Das bereitet mir nun mehr Probleme, da ich hier jetzt nichtw eiss wie ich vorgehen soll. Gibt es da bestimmte Schema F Ideen?
klarsoweit Auf diesen Beitrag antworten »
RE: Extrema mit Nebenbedingung
Also aus folgt für x ungleich Null .

Wie gesagt: wie folgt daraus xy^1=-1 ? verwirrt

kannst du bequem nach lambda umstellen.
HBX8X Auf diesen Beitrag antworten »

Ups, ich meinte genau das was du meintest. Du gehst aber anscheinend von dem Fall x ungleich Null aus. Ich bin darauf wie folgt gekommen. Ich habe bei der ersten Komponente des Gradienten die 2x ausgeklammert -> 2x(lambda*y^2 +1)=0. Und nun kann man doch den Satz des Nullprodukts anwenden, d.h.

1. Fall: 2x=0
2. Fall: lambda*y^2 +1=0.

Ich komm nur auf die zwei Gleichungen. Wie kommst du auf x ungleich 0 bzw. was mach ich falsch?

Gruß & vielen dank für die Hilfe!

Edit: Ich vermuet mal bei den zweiten Fall sagt man das x ungleich 0 ist, also anscheinend habe ich es doch korrekt gemacht? Aber wieso sagt man x ungleich 0? (Habe das wohl unbewusst immer so gemacht bisher).
klarsoweit Auf diesen Beitrag antworten »

Zitat:
Original von HBX8X
Wie kommst du auf x ungleich 0 bzw. was mach ich falsch?

Eigentlich nichts. Der 2. Fall ist doch nur relevant, wenn der 1. Fall nicht eingetreten ist, also x ungleich Null ist. Augenzwinkern
 
 
HBX8X Auf diesen Beitrag antworten »

Da ist irgendetwas glaube ich ziemlich außer kontrolle gelaufen. Freude
HBX8X Auf diesen Beitrag antworten »

Eigentlich fällt mir wirklich kein Fehler auf, deshalbf rag ich lieber noch einmal, ist das so richtig bzgl. den Lösungen? verwirrt

Gruß!
klarsoweit Auf diesen Beitrag antworten »

Hm. Müßte das nicht zwischendrin heißen? verwirrt
HBX8X Auf diesen Beitrag antworten »

Ok, danke da ist mir dann wohl ein Fehler unterlaufen. Kurze Frage:

Wenn ich jeweils lambda,x und y bestimmt habe wie sieht dann mein Extrempunkt aus und ist überhaupt Lambda wichtig dafür? (x,y) wäre mein Extremum oder? Und das Lambda kann ich halt ignorieren nachdem ich x und y bestimmt habe.

Ich soll danach die Funktionswerte f(xi) berechnen an den kritischen Punkten der ersten Teilaufgabe. Also einfach den kritischen Punkt einsetzen, dafür bekomm ich dann jeweils aber nur einen Punkt herraus. Wieso nur einen? Sagt das etwas über die Art des Extremum aus? Denn die nächste Teilaufgabe wäre dies nämlich.
klarsoweit Auf diesen Beitrag antworten »

Also das Verfahren liefert nur die kritischen Punkte. Zur Beantwortung der Frage, ob da tatsächlich ein Extremum ist (und welche Art), müssen weitere Überlegungen angestellt werden.
HBX8X Auf diesen Beitrag antworten »

Genau und normalerweise ist dasdoch die Hesse Matrix? In Teilaufgabe (b) wird aber nur gesagt (nach dem man die krit. Punkte berechnet hat)

(b)Berechnen sie die Funktionswerte f(Vektor(x)) an den kritischen Punkten aus a. Welche Vermutung über die Art der kritischen Punkte legt ihr Ergebnis nahe?

Ich denke das soll definitiv ohne Hesse Matrix gelöst werden. Und hier weiss ich nicht wie man das machen könnte nach dem man die Funktionswerte bestimmt.

Und nochmal, da ich es noch nicht ganz verstanden habe. Werden die berechneten Lambdas mit in meine krit. Punkte beachtet oder sieht das Allgemeine krit. Punkt wie folgt aus (x,y) , also nicht (x,y,Lambda). Lambda dient nur zur Berechnung von x und y, da man dafür sonst bzgl. der NB keine Lösungsmenge bekommt richtig ?
klarsoweit Auf diesen Beitrag antworten »

Die kritischen Punkte haben die Form (x,y). Das lambda diente ja nur zur Berücksichtigung der Nebenbedingung. Und gerade wegen der Nebenbedingung funktioniert ja auch nicht das übliche Verfahren mit der Hesse-Matrix.

Also bestimme mal die kritischen Punkte und die Funktionswerte.
HBX8X Auf diesen Beitrag antworten »

Naja mir ist da noch die geränderte Hesse Matrix bekannt. :P Ich denke das sollen wir hier aber nicht machen, sondern wir sollen viel mehr die Funktion betachten und daraus rückschlüsse ziehen.

Noch eine letzte Frage: Wieso bekomm ich nur einen Punkt herraus wenn ich den Extremum (x,y) in meine Ausgangsfunktion einsetze?
klarsoweit Auf diesen Beitrag antworten »

Zu jedem kritischen Punkt bekommst du einen Funktionswert in R. (Die Funktion bildet auf R ab, nicht auf R², wie es fälschlicherweise in deinem ersten Post steht.)
Leopold Auf diesen Beitrag antworten »

Im Anhang eine Visualisierung mit dynamischer Geometrie. Zum Öffnen der Datei installiere Euklid.
HBX8X Auf diesen Beitrag antworten »

So habe insgesamt 10 Kritische Punkte herraus, wobei 4 x-Werte komplex sind und ihr zugehöriger y Wert 1/0 ist und somit nicht definiert ist. Die anderen 6 teilen sich wie folgt auf:

3 Stück bei (0,1) wobei der Funktionswert für diese drei alle -1 ergibt.
3 Stück bei (0,-1) wobei der Funktionswert für diese drei alle 1 ergibt.

Ist das richtig so ? Und welche Rücksclüsse auf die Art kann ich nun daraus entnehmen.

Edit 1: @Leopold. Danke schön, werde es mir ansehen wenn ich wieder zuhause bin! Leider habe ich gerade nicht die Mittel dazu die Datein zu öffnen.

Gruß.

Edit 2: Ich geh mal davon aus, dass bei (0,1) alles minima sind und bei (0,-1) alles maxima sind. Big Laugh
Leopold Auf diesen Beitrag antworten »

@ "alte Mathe-Hasen"

Ich kenne keine Rechenaufgabe zu relativen Extrema mit Nebenbedingungen, die sich ohne Lagrangesche Multiplikatoren nicht ebenso gut oder sogar besser lösen läßt als mit ihnen. Wer von euch kennt ein Gegenbeispiel?


@ HBX8X

Ganz ohne Lagrangesche Multiplikatoren.
Zunächst löst man die Nebenbedingung auf:



Die Auflösung nach ist nur möglich, falls gilt, denn genau dann ist die rechte Seite der letzten Gleichung nichtnegativ. Für solche setzen wir in ein. Zu untersuchen ist somit die Funktion



Für die Ableitung gilt:



Der Term für wird nur für , was aber nicht im Definitionsbereich von liegt. Somit besitzt keine Nullstellen, womit in den Intervallen und streng monoton ist. Das Verhalten von für zeigt, daß in streng monoton wächst und in streng monoton fällt. Bei befinden sich daher Randminima. Der zugehörige -Wert aus der Nebenbedingung ist beide Male .

Zusammenfassend gilt: besitzt unter der Nebenbedingung bei ein relatives Minimum mit Wert und bei ein relatives Minimum mit Wert . Weitere Extremwerte existieren nicht.
HBX8X Auf diesen Beitrag antworten »

Also sind es zwei relative Minima? Und was sagt dann uns der Funktionswert aus? Big Laugh Ich habe bei dem Gleichungssystem meherer gleichwertige Lösungen herrausbekommen. Das ist doch richtig ?
Leopold Auf diesen Beitrag antworten »

Das erscheint merkwürdig. Wie kann es zwei Minima geben ohne Maximum dazwischen? Nun, es liegt daran, daß nicht zusammenhängend ist (siehe Euklid-Zeichnung, ziehe dort an den Punkten).

Deine Rechnung habe ich mir nicht angeschaut. Das ist mir viel zu kompliziert. Wenn es auch mit einer Variablen geht, warum soll ich dann drei nehmen?
HBX8X Auf diesen Beitrag antworten »

Also hast du doch von den Funktionswerten Rückschlüsse auf die Art des Extremum geschlossen?
Leopold Auf diesen Beitrag antworten »

Hier steht die vollständige Begründung:

Zitat:
Original von Leopold
Der Term für wird nur für , was aber nicht im Definitionsbereich von liegt. Somit besitzt keine Nullstellen, womit in den Intervallen und streng monoton ist. Das Verhalten von für zeigt, daß in streng monoton wächst und in streng monoton fällt. Bei befinden sich daher Randminima.
Neue Frage »
Antworten »



Verwandte Themen

Die Beliebtesten »
Die Größten »
Die Neuesten »